Can someone explain this equality to me (complex variables)

Click For Summary
The discussion revolves around evaluating an indefinite integral involving complex variables, specifically questioning the inequality |e^{-3z}|=e^{-3y}≤1. The user struggles to understand how this inequality holds true, given the transformation of the complex exponential function. They clarify that for points on the semicircular path, y is non-negative, leading to the conclusion that e^{-3y} must be less than or equal to 1. The key point is that since e^{-3y} is always positive and decreases as y increases, it remains bounded by 1 on the specified path. This understanding is crucial for correctly evaluating the integral.
richyw
Messages
179
Reaction score
0

Homework Statement



I hate to upload the whole problem, but I am trying to evaluate an indefinite integral, and I can follow the solution until right near the end. The example says that for a point on C_R|e^{-3z}|=e^{-3y}\leq 1. I don't understand how they can say this. Below is the question, with a drawing of the region. I have highlighted the step that I do not understand.

http://media.newschoolers.com/uploads/images/17/00/70/52/45/705245.png

Homework Equations



The Attempt at a Solution



I might be missing something easy, but I can't see how this is true!
 
Last edited by a moderator:
Physics news on Phys.org
##e^{3iz}=e^{3i(x+iy)}=e^{-3y+3ix}=e^{-3y}e^{3ix}## and ##|e^{i3x}|=1##.
 
exp(3iz) = exp(3ix - 3y) = exp(-3y) exp(3ix)
The magnitude of this complex number is |exp(-3y)| times 1, because exp(3ix) = cos(3x) + i sin(3x), and |exp(3ix)| is the sum of a squared cosine and a squared sine of the same argument. And then of course |exp(-3y)| = exp(-3y) because e-to-the-anything is always positive.

Why is exp(-3y) <= 1? Because exp(0) = 1, and on the given semicircular path, y is non-negative.
 
  • Like
Likes 1 person
Question: A clock's minute hand has length 4 and its hour hand has length 3. What is the distance between the tips at the moment when it is increasing most rapidly?(Putnam Exam Question) Answer: Making assumption that both the hands moves at constant angular velocities, the answer is ## \sqrt{7} .## But don't you think this assumption is somewhat doubtful and wrong?

Similar threads

Replies
4
Views
2K
  • · Replies 14 ·
Replies
14
Views
3K
  • · Replies 11 ·
Replies
11
Views
2K
  • · Replies 6 ·
Replies
6
Views
3K
Replies
5
Views
2K
  • · Replies 17 ·
Replies
17
Views
2K
  • · Replies 14 ·
Replies
14
Views
2K
  • · Replies 1 ·
Replies
1
Views
2K
  • · Replies 22 ·
Replies
22
Views
2K
Replies
8
Views
4K